Skip to content

Q1

← Back

Basic Info

Probability
└── Homework
    └── W4
        └── Q1.tex

Preview

\documentclass[12pt]{article}

\usepackage{graphicx}
\usepackage{amssymb}
\usepackage{amsmath}
\usepackage[margin=1in]{geometry}
\usepackage{fancyhdr}
\usepackage{enumerate}
\usepackage[shortlabels]{enumitem}

\pagestyle{fancy}
\fancyhead[l]{Li Yifeng}
\fancyhead[c]{Homework \#4}
\fancyhead[r]{\today}
\fancyfoot[c]{\thepage}
\renewcommand{\headrulewidth}{0.2pt}
\setlength{\headheight}{15pt}

\newcommand{\bP}{\mathbb{P}}

\begin{document}

    \section*{Question 1}

    \noindent A student is applying to Harvard and Dartmouth. She estimates that she has a probability of $0.5$ of being accepted at Dartmouth and $0.3$ of being accepted at Harvard. She further estimates that the probability that she will be accepted by both is $0.2$.

    \bigskip

    \begin{enumerate}[start=1,label={\bfseries Part \arabic*:},leftmargin=0in]
        \bigskip\item What is the probability that she is accepted by Dartmouth if she is accepted by Harvard?

        \subsection*{Solution}

            Let $\it{accept}$ and $\it{reject}$ denote the results of an application, and let $D$ and $H$ denote the events of being accepted by Dartmouth and by Harvard, respectively. Define the probability space as

            \[
            \begin{aligned}
                \Omega &= \{\mathrm{accept},\,\mathrm{reject}\}\\
                \mathcal{F} &= \mathcal{P}(\Omega)\\
                \bP &:\enspace \bP(D) = 0.5,\enspace \bP(H) = 0.3
            \end{aligned}
            \]

            We know that

            \[\bP(D\cap H) = 0.2\]

            Then we have

            \[\bP_H(D) = \frac{\bP(D\cap H)}{\bP(H)} = \frac{2}{3} \approx 0.667\]

        \subsection*{Answer}

            \[\boxed{\bP_H(D) = \frac{2}{3} \approx 0.667}\]

        \bigskip\item Is the event “accepted at Harvard” independent of the event “accepted at Dartmouth”?

        \subsection*{Solution}

            Applying the Bayes’ Theorem, we know that

            \[\bP_D(H) = \frac{\bP_H(D)\bP(H)}{\bP(D)} = 0.4 \ne 0.3 = \bP(H)\]

            Then we know that $D$ and $H$ are not stochastically independent.

        \subsection*{Answer}

            \[\boxed{\text{No, $H$ is not stochastically independent of $D$.}}\]

    \end{enumerate}

\end{document}